Administración     

Olimpiadas de Matemáticas
Página de preparación y problemas

Selector
La base de datos contiene 1154 problemas y 775 soluciones.
OME Local
OME Nacional
OIM
OME Andalucía
Retos UJA
Problema 592
Una empresa produce semanalmente $300$ bicicletas de montaña que vende íntegramente al precio de $600$ euros cada una. Tras un análisis de mercados observa que si varía el precio, también varían sus ventas (de forma continua) según la siguiente proporción: por cada $7$ euros que aumente o disminuya el precio de sus bicicletas, disminuye o aumenta la venta en $3$ unidades, respectivamente.
  1. ¿Puede aumentar el precio y obtener mayores ingresos?
  2. ¿A qué precio los ingresos serán máximos?
pistasolución 1info
Pista. Obtén los ingresos como función del precio y observa que es una función cuadrática. Analiza cuándo es creciente/decreciente y cuándo alcanza su valor máximo/mínimo.
Solución. Sea $i(p)$ la función que devuelve los ingresos en términos del precio $p$ por unidad. El número de bicicletas vendidas en función de $p$ viene dado por $\frac{3900-3p}{7}$ (es una función lineal de la forma $ap+b$, donde $a=-\frac{3}{7}$ ya que debe aumentar/disminuir $3$ unidades si $p$ disminuye/aumenta $7$ euros y $b$ se ajusta para que $600a+b=300$). De esta manera, tenemos que $i(p)=\frac{3900-3p}{7}p$ es igual al número de unidades vendidas multiplicado por el precio de la unidad.

Podemos completar el cuadrado para expresar \[i(p)=\tfrac{-3}{7}(p^2-1300p)=\tfrac{3}{7}650^2-\tfrac{3}{7}(p-650)^2.\] Por tanto, los ingresos serán máximos cuando $(p-650)^2$ sea mínimo, es decir, para $p=650$, en cuyo caso los ingresos máximos vendrán dados por $\tfrac{3}{7}650^2$ euros, respondiendo así al apartado (b). En cuanto al apartado (a), la respuesta es afirmativa puesto que la función $i(p)$ es creciente en el intervalo $(0,650)$ y, en particular, en el precio inicial $p=600$.

Nota. La última parte se puede analizar también con la derivada. Probablemente, el ejercicio original estaba pensado para hacerse con una derivada.

Si crees que el enunciado contiene un error o imprecisión o bien crees que la información sobre la procedencia del problema es incorrecta, puedes notificarlo usando los siguientes botones:
Informar de error en enunciado Informar de procedencia del problema
Problema 591
¿Qué dígitos se han omitido en la siguiente multiplicación? \[\begin{matrix} &2&*&*\\ \times&&*&*\\\hline &*&6&1\\ *&*&*&\\\hline *&*&0&1 \end{matrix}\]
pistasolución 1info
Pista. Deduce las cifras una a una y separa algún caso cuando haya más de una posibilidad.
Solución. Deduciendo los dígitos de uno en uno, se llega fácilmente a la siguiente solución única: \[\begin{matrix} &2&8&7\\ \times&&2&3\\\hline &8&6&1\\ 5&7&4&\\ \hline 6&6&0&1 \end{matrix}\]
Si crees que el enunciado contiene un error o imprecisión o bien crees que la información sobre la procedencia del problema es incorrecta, puedes notificarlo usando los siguientes botones:
Informar de error en enunciado Informar de procedencia del problema
Problema 589
Encontrar todas las funciones $f:\mathbb{R}\to\mathbb{R}$ tales que $$f(x + f(y + f(x + f(y + f(x))))) = 3x + 2y$$ para cualesquiera números reales $x,y\in\mathbb{R}$.
pistasolución 1info
Pista. Demuestra que existe $x_0\in\mathbb{R}$ tal que $f(x_0)=0$ y sustituye $x=x_0$ en la ecuación funcional.
Solución. El miembro de la derecha toma todos los valores reales posibles, luego está claro que $f$ debe ser sobreyectiva. En particular, existe $x_0\in\mathbb{R}$ tal que $f(x_0)=0$ (por ejemplo, para $x=y=0$, podríamos tomar $x_0=f(f(f(f(0))))$ y cumpliría que $f(x_0)=0$). Sustituyendo $x=x_0$ en la ecuación nos quedaría \[f(x_0+f(y+f(x_0+f(y))))=3x_0+2y.\] Sumando $y$ a ambos miembros y volviendo a aplicar $f$, tenemos que \[f(y+f(x_0+f(y+f(x_0+f(y)))))=f(y+3x_0+2y)=f(3y+3x_0).\] Ahora bien, al miembro de la izquierda en esta igualdad se le puede aplicar la ecuación funcional del enunciado (cambiando $x\mapsto y$ y $y\mapsto x_0$). Esto nos dice que \[3y+2x_0=f(y+f(x_0+f(y+f(x_0+f(y)))))=f(y+3x_0+2y)=f(3y+3x_0).\] Finalmente, haciendo el cambio $t=3y+3x_0$, obtenemos que \[f(t)=t-x_0,\qquad\text{para todo }t\in\mathbb{R}.\] Sin embargo, esta función cumple la ecuación inicial solo cuando $x_0=0$, de donde deducimos que la identidad $f(t)=t$ para todo $t\in\mathbb{R}$ es la única solución al problema.
Si crees que el enunciado contiene un error o imprecisión o bien crees que la información sobre la procedencia del problema es incorrecta, puedes notificarlo usando los siguientes botones:
Informar de error en enunciado Informar de procedencia del problema
Problema 587
Determinar todas las funciones $f:\mathbb{R}\to\mathbb{R}$ tales que $$f(xf(y) + y) = f(xy) + f(y)$$ para cualesquiera números reales $x,y\in\mathbb{R}$.
Sin pistas
Sin soluciones
info
Si crees que el enunciado contiene un error o imprecisión o bien crees que la información sobre la procedencia del problema es incorrecta, puedes notificarlo usando los siguientes botones:
Informar de error en enunciado Informar de procedencia del problema
Problema 577
Sea $a>1$ un número real. Encontrar todas las soluciones de la ecuación $$\sqrt{a-\sqrt{a+x}} = x$$ en términos de $a$.
pistasolución 1info
Pista. Eleva al cuadrado dos veces para eliminar raíces, pero con cuidado de si los radicandos son positivos o negativos en función de $a$.
Solución. Elevando al cuadrado, tenemos que $a-\sqrt{a+x}=x^2$ y depejamos la raíz como $x^2-a=\sqrt{a+x}$. Elevando de nuevo al cuadrado, tenemos la ecuación \[x^4-2ax^2+a^2=a+x\ \Leftrightarrow\ x^4-2ax^2+x+a(a-1)=0.\] Esta última ecuación de grado $4$ puede parecer imposible de resolver ya que tiene el parámetro $a$, pero resulta que se puede factorizar como producto de dos polinomios de grado $2$: \[0=x^4-2ax^2+x+a(a-1)=(x^2-x-a)(x^2+x-a+1).\] Esto nos da dos ecuaciones de segundo grado. La primera ecuación nos da dos soluciones: \[x^2-x-a=0\ \leadsto\ x_1=\frac{1+\sqrt{1+4a}}{2},\ x_2=\frac{1-\sqrt{1+4a}}{2}.\] La segunda por su parte nos da otras dos soluciones: \[x^2+x-a+1=0\ \leadsto\ x_3=\frac{-1+\sqrt{4a-3}}{2},\ x_4=\frac{-1-\sqrt{4a-3}}{2}.\] Sin embargo, tenemos que comprobar si cada una de estas cuatro soluciones realmente cumple la ecuación original (al elevar al cuadrado podemos haber introducido soluciones ficticias (donde las raíces del enunciado no están definidas o no dar los valores correctos).
  • $x_2$ y $x_4$ son negativos, luego no son soluciones ya que $x=\sqrt{a-\sqrt{x+a}}\geq 0$ nos dice que $x$ tiene que ser positivo (una raíz nunca es negativa).
  • $x_1$ tampoco es solución ya que $x_1=\frac{1+\sqrt{1+4a}}{2}\gt\frac{\sqrt{4a}}{2}=\sqrt{a}$ y cualquier solución cumple que $x=\sqrt{a-\sqrt{x+a}}\lt\sqrt{a}$.
  • $x_3$ sí que es solución (la nota de abajo nos da otro motivo del porqué). Para verlo, nos damos cuenta primero de que $x_3\geq 0$ y, por tanto, $x_3+a\geq 0$; además, tenemos que \begin{align*} 0\leq x_3+a&=\frac{2a-1+\sqrt{4a-3}}{2}\\ &\leq \frac{2a-1+2a-1}{2}=2a-1=a^2-(a-1)^2\leq a^2. \end{align*} Por tanto, se cumple que $\sqrt{x_3+a}\leq a$ y la raíz grande del enunciado también está bien definida. Deducimos que $x_3$ es la única solución.

Nota. Un atajo que nos puede hacer entender mejor el problema es darse cuenta de que el miembro de la izquierda $f(x)=\sqrt{a-\sqrt{a+x}}$ es una función continua y estrictamente decreciente definida en un intervalo $[0,x_0]$ con $f(0)>0$ y $f(x_0)=0$ y que el de la derecha $g(x)=x$ es también continua y estrictamente creciente con $g(0)=0$ y $\lim_{x\to\infty}g(x)=+\infty$, luego la ecuación $f(x)=g(x)$ tiene necesariamente una única solución.

Si crees que el enunciado contiene un error o imprecisión o bien crees que la información sobre la procedencia del problema es incorrecta, puedes notificarlo usando los siguientes botones:
Informar de error en enunciado Informar de procedencia del problema
José Miguel Manzano © 2010-2024. Esta página ha sido creada mediante software libre